Search found 14 matches


> total permutations of = 6P2 =15
Samir,
Not that this number is used in the problem, but 6P2 is 30, not 15.

by naren_nayak

Thu Nov 08, 2007 8:41 am
Forum: Problem Solving
Topic: SET 28 : Good prob. quest....but confusing OA...PL.TRY
Replies: 10
Views: 2286

Stacey, A minor mistake towards the end... > 300,000 > C > So the costs are at least 300,000. So profit is at most 200,000, but they could be below or above 150,000. Can't tell. Answer is E. C < 300,000 (Your last step assumed the opposite) Profit = 500,000 - 300,000 = 200,000 Profits are 200k in th...

by naren_nayak

Sat Nov 03, 2007 10:41 am
Forum: Data Sufficiency
Topic: need help on ds qs from gmatprep1
Replies: 5
Views: 3924

4 & 25 have an LCM of 100.
And so do 20 & 50.
The former is divisible by 8, the latter is not.

by naren_nayak

Thu Nov 01, 2007 8:48 am
Forum: Data Sufficiency
Topic: GMAT Powerprep -Number properties question
Replies: 3
Views: 1728

Since std dev = 6 => 1 std dev = 6, 2 std dev = 12.
1 std dev below the mean (which is 21) is 21-6 = 15 hours
2 std dev below the mean = 21 - 12 = 9 hours
Time Pat watched TV is between 9 and 15 hours. 12 is the only answer that falls in this range. Hence D.

by naren_nayak

Wed Oct 31, 2007 8:05 pm
Forum: Problem Solving
Topic: GMATPrep Test 1 : PS #32
Replies: 2
Views: 1639

I don't know if this is the easiest method, but I try to focus just on being compliant with the original conditions with values that go against the conditon mentioned in the individual questions. In this case, the conditions given are that d is positive & 0 < 1 - (c/d) < 1 i.e. 0 < (d-c)/d < 1 F...

by naren_nayak

Sun Oct 28, 2007 9:54 am
Forum: Problem Solving
Topic: How to pick values for such problems
Replies: 2
Views: 2109

> so total is 6! * 7! Samir, Shouldn't that be 7! * 7! ? The first seat you considered for the 1st woman can take 1 of 7 women so 7 * 6! The only configuration in which 7 men and 7 women can be seated around a table so that no two women are seated next to each other is to alternate, as Samir said. M...

by naren_nayak

Tue Oct 23, 2007 9:59 am
Forum: Problem Solving
Topic: Tough GMAT PS question
Replies: 6
Views: 2403

Bob just filled his car's gas tank with 20 gallons of gasohol, a mixture consisting of 5% ethanol and 95% gasoline. If his car runs best on a mixture consisting of 10% ethanol and 90% gasoline, how many gallons of ethanol must he add into the gas tank for his car to achieve optimum performance? 20 g...

by naren_nayak

Mon Oct 22, 2007 6:36 am
Forum: Problem Solving
Topic: MGMAT CAT 600-700 ratio question
Replies: 3
Views: 2959

Adding to Samir's answer:

> 2). S2=43
>similarly 633 =43 +11x
> i.e 590/11 produces a remainder hence this is not a term

For 2, 633 could be part of the S1, S3, S5... sequence, and since S1 is not specified, 2 is insufficient.

by naren_nayak

Tue Oct 16, 2007 9:33 am
Forum: Data Sufficiency
Topic: Sequence
Replies: 2
Views: 2555

I think the answer is C. a) You could have x=1, y=4: 1 2 4 5 5 8 => Range = 7, which is < 9 OR x=9, y=13: 2 4 5 8 9 13 => Range = 10, which is > 9 Hence A is insufficient. b) You could have x=8,y=12: 2 4 5 8 8 12 => Range = 10, which is > 9 OR x=8, y=9: 2 4 5 8 8 9 => Range = 7, which is < 9 Hence B...

by naren_nayak

Tue Oct 16, 2007 9:19 am
Forum: Data Sufficiency
Topic: Range...
Replies: 1
Views: 1466

So dogged were Frances Perkins’ investigations of the garment industry, and her lobbying for wage and hour reform was persistent, Alfred E. Smith and Franklin D. Roosevelt recruited Perkins to work within the government, rather than as a social worker. A. and her lobbying for wage and hour reform wa...

by naren_nayak

Sat Oct 06, 2007 8:03 pm
Forum: Sentence Correction
Topic: Brutal SC - SET 27 Q17
Replies: 11
Views: 2453

I second that. B is the only choice with the correct idiomatic use of "perceived...to be".

by naren_nayak

Fri Sep 28, 2007 10:08 am
Forum: Sentence Correction
Topic: Brutal SC #49
Replies: 11
Views: 6876

One could also use the process of elimination here: (54,820)^2 should have a 0 in the unit's position & (54,822)^2 should end with a 4. The average of these two numbers will have a 2 in the unit's position. That eliminates A, B, C & E (A ends with a 1, B & C end with a .25 and E ends wit...

by naren_nayak

Sun Sep 16, 2007 8:21 am
Forum: Problem Solving
Topic: Manhattan Challenge question
Replies: 7
Views: 1897

OG 11th edition, practice question #125

If r & s are positive integers, is r/s an integer? (1) Every factor of s is also a factor of r (2) Every prime factor of s is also a prime factor of r OA is A. I don't get it. Isn't the following possible? Scenario 1: r = 4 = 1*2*2, s = 2 = 1*2; r/s = 4/2 => r/s is an integer Scenario 2: r = 5 =...

by naren_nayak

Fri Jul 06, 2007 9:45 am
Forum: Data Sufficiency
Topic: OG 11th edition, practice question #125
Replies: 11
Views: 2687